Question

In: Economics

Suppose there are two firms in a market who each simultaneously choose a quantity. Firm 1’s quantity is q1, and firm 2’s quantity is q2.


Suppose there are two firms in a market who each simultaneously choose a quantity. Firm 1’s quantity is q1, and firm 2’s quantity is q2. Therefore the market quantity is Q = q1 + q2. The market demand curve is given by P = 160 - 2Q. Also, each firm has constant marginal cost equal to 10. There are no fixed costs.

The marginal revenue of the two firms are given by:

  • MR1 = 160 – 4q1 – 2q2

  • MR2 = 160 – 2q1 – 4q2.

A) How much output will each firm produce in the Cournot equilibrium and what will be the market price of the good?

B) What is the deadweight loss that results from this duopoly?

C) How much profit does each firm make?

D) Suppose Firm 2 produced 30 units of output. How much output should Firm 1 produce in order to maximize profit?

Solutions

Expert Solution

Each firm’s marginal cost function is MC = 10 and
the market demand function is
=> P = 160 – 2Q
=> P = 160 - 2(q1+q2)

Finding best response functions for both firms:

Revenue for firm 1
R1 = P*q1 = (160 - 2(q1+q2))*q1 = 160q1 – 2q12 – 2q1q2.

Firm 1 has the following marginal revenue and marginal cost functions:
MR1 = 160 – 4q1 – 2q2
MC1 = 10

Profit maximization implies:
MR1 = MC1
160 – 4q1 – 2q2 = 10
which gives the best response function:
q1 = 37.5 - 0.5q2.

By symmetry, Firm 2’s best response function is:
q2 = 37.5 - 0.5q1.

Cournot equilibrium is determined at the intersection of these two best response functions:
q1 = 37.5 - 0.5(37.5 - 0.5q1)
q1 = 18.75 + 0.25q1
q1 = 25

A) This gives q1 = q2 = 25 units This the Cournot solution.
     Price is (160 – 2*50) = $60

B) deadweight loss = 0.5*(current price - competitive price)*(competitive quantity - combinded cournot quantity)

Competition has P = MC
160 - 2Q = 10
150 = 2Q
Q = 75 and so P = 10

dead weight loss = 0.5*(60 - 10)*(75 - 50) = $625

C) Profit to each firm = (60 – 10)*25 = $1250

D) Suppose Firm 2 produced 30 units of output.
     Firm 1 produce q1 = 37.5 - 0.5q2 or q1 = 37.5 - 0.5*30 = 22.5 units (rounding to 23 units) in order to maximize profit


Related Solutions

Suppose there are two firms in a market who each simultaneously choose a quantity. Firm 1’s...
Suppose there are two firms in a market who each simultaneously choose a quantity. Firm 1’s quantity is q1, and firm 2’s quantity is q2. Therefore the market quantity is Q = q1 + q2. The market demand curve is given by P = 100 – 2Q. Also, each firm has constant marginal cost equal to 10. There are no fixed costs. The marginal revenue of the two firms are given by: MR1 = 100 – 4q1 – 2q2 MR2...
Suppose there are two firms in a market who each simultaneously choose a quantity. Firm 1’s...
Suppose there are two firms in a market who each simultaneously choose a quantity. Firm 1’s quantity is q1, and firm 2’s quantity is q2. Therefore the market quantity is Q = q1 + q2. The market demand curve is given by P = 225 - 3Q. Also, each firm has constant marginal cost equal to 9. There are no fixed costs. The marginal revenue of the two firms are given by: MR1 = 225 – 6q1 – 3q2 MR2...
Suppose there are two firms in a market who each simultaneously choose a quantity. Firm 1’s...
Suppose there are two firms in a market who each simultaneously choose a quantity. Firm 1’s quantity is q1, and firm 2’s quantity is q2. Therefore the market quantity is Q = q1 + q2. The market demand curve is given by P = 160 - 2Q. Also, each firm has constant marginal cost equal to 10. There are no fixed costs. The marginal revenue of the two firms are given by: MR1 = 160 – 4q1 – 2q2 MR2...
Suppose there are two firms in a market who each simultaneously choose a quantity.Firm 1's quantity...
Suppose there are two firms in a market who each simultaneously choose a quantity.Firm 1's quantity is q1, and firm 2's quantity is q2. Therefore the market quantity is Q= q1+q2. The market demand curve is given by P=130-Q. Also, each firm has constant marginal cost equal to 25. There are no fixed costs. Marginal revenue of the firms are given by MR1=130-2q1-q2 & MR2=130-q1-2q2. A) How much output will each firm produce in the Cournot equilibrium ? B) What...
Suppose there are two firms in a market who each simultaneouslychoose a quantity. Firm 1’s...
Suppose there are two firms in a market who each simultaneously choose a quantity. Firm 1’s quantity is q1, and firm 2’s quantity is q2. Therefore the market quantity is Q = q1 + q2. The market demand curve is given by P = 100 – 4Q. Also, each firm has constant marginal cost equal to 28. There are no fixed costs.The marginal revenue of the two firms are given by:MR1 = 100 – 8q1 – 4q2MR2 = 100 –...
Player 1 and Player 2 (independently and simultaneously) choose quantities, q1 and q2. The cost of...
Player 1 and Player 2 (independently and simultaneously) choose quantities, q1 and q2. The cost of producing qi units is c(qi) = qi2 ; and the demand curve is given by P(Q) = 5 − Q: (i.e., if Player 1 produces q1 and Player 2 produces q2; each sells all his units at price 5 − q1 − q2. Find the profit.
two firms produce a homogenous good, each firm simultaneously decides on its quantity to produce and...
two firms produce a homogenous good, each firm simultaneously decides on its quantity to produce and respectively letting the market determine the price. the market price for both firms is   P= 4020-10(q1+q2). firm 1 faces marginal cost c1=40. firm faces marginal cost c2=80. 11.1 write down the profit function for each firm as a function of q1 and q2 .2 Determine the best response function of each firm and draw a graph of the best response curves .3 determine the quantities...
Suppose there are only two firms in the market, firm 1 and firm 2. They produce...
Suppose there are only two firms in the market, firm 1 and firm 2. They produce identical products. Firm 1 has a constant marginal cost where AC1 =MC1 =20, and firm 2 has a constant marginal cost AC2 =MC2 =8. The market demand function is given by Q = 100 - 0.5P. a) Find the Cournot Nash Equilibrium price and quantity, write down the profits for each firm. (Use "q1" to represent output level for firm 1, and "profit1" to...
Two firms, Firm 1 and Firm 2, compete by simultaneously choosing prices. Both firms sell an...
Two firms, Firm 1 and Firm 2, compete by simultaneously choosing prices. Both firms sell an identical product for which each of 100 consumers has a maximum willingness to pay of $40. Each consumer will buy at most 1 unit, and will buy it from whichever firm charges the lowest price. If both firms set the same price, they share the market equally. Costs are given by ??(??)=16??ci(qi)=16qi. Because of government regulation, firms can only choose prices which are integer...
Assume two firms 1 and 2. The inverse market demand function is given by:              P=30-(q1+q2)...
Assume two firms 1 and 2. The inverse market demand function is given by:              P=30-(q1+q2) Each firm produces with marginal costs of MC = 6 Fixed costs are zero. The next questions refer to the Cournot duopoly. Question 1 (1 point) What is Firm 1's total revenue function? Question 1 options: TR1=30q1 -q1 -q22 TR1=30-2q1-q2 TR1=30q1 -q12-q2 None of the above. Question 2 (1 point) What is Firm 1's marginal revenue function? Question 2 options: MR1=30-2q1 -q2 MR1=30-q1-2q2 MR1=30-2q1-2q2...
ADVERTISEMENT
ADVERTISEMENT
ADVERTISEMENT